Assume​ short-run production. Indicate whether the statement below is true​ (T) or false​ (F). nothing​: The difference between the total cost and the total variable cost is a constant. nothing​: When total cost or total variable cost is​ increasing, there are increasing marginal returns to the variable input. nothing​: Changes in fixed costs do not affect the shape or placement of the total cost curve. nothing​: The marginal cost is the slope of the total cost curve or the total variable cost curve. nothing​: The average cost curve is everywhere above the average variable cost curve. nothing​: The marginal cost at a particular output level is the slope of a line from the origin to the corresponding point on the cost curve.

Answers

Answer 1

Answer:

1. The difference between the total cost and the total variable cost is a constant. - TRUE

The difference between the 2 is indeed constant and is the Total Fixed cost which does not change throughout the production process.

2. When total cost or total variable cost is​ increasing, there are increasing marginal returns to the variable input. - FALSE

With only the total cost or total variable cost given, it is not possible to tell how the Marginal returns to the input is faring.

3. Changes in fixed costs do not affect the shape or placement of the total cost curve. - FALSE

Fixed costs are part of the total cost curve so if they change they will impart the total cost curve. An increase may not change the shape but it will definitely change the placement of the Total cost curve.

4. The marginal cost is the slope of the total cost curve or the total variable cost curve. - TRUE

The slope of either the Total cost or variable cost curves are the graphical representations of a change in either which is the definition of the Marginal cost.

5. The average cost curve is everywhere above the average variable cost curve. - TRUE

As the average cost is the sum of both the average fixed and average variable costs, it will always be higher than either so it is higher than the Average variable cost.

6. The marginal cost at a particular output level is the slope of a line from the origin to the corresponding point on the cost curve. - FALSE

Marginal cost measures the difference in cost from one unit to the next. A line from the origin to the corresponding point would have measured for all units produced making it the Average cost not Marginal Cost.


Related Questions

Ten years ago you put $150000.00 into an interest earning account. Today it's worth $275000. What is the effective annual interest earned on the account

Answers

Answer:

the effective annual interest earned on the account is 6.25%.

Explanation:

The effective annual interest earned on the account can be calculated as follows :

PV = - $150,000

N = 10

PMT = $0

P/yr = 1

FV = $275,000

R = ?

Using a Financial calculator, the  effective annual interest, R, earned on the account will be : 6.2488 or 6.25%.

Praveen Co. manufactures and markets a number of rope products. Management is considering the future of Product XT, a special rope for hang gliding, that has not been as profitable as planned. Since Product XT is manufactured and marketed independently of the other products, its total costs can be precisely measured. Next year’s plans call for a $350 selling price per 100 yards of XT rope. Its fixed costs for the year are expected to be $315,000, up to a maximum capacity of 550,000 yards of rope. Forecasted variable costs are $245 per 100 yards of XT rope.
Required:
1. Estimate Product XT's break-even point in terms of (a) sales units and (b) sales dollars.
2. Prepare a CVP chart for Product XT. Use 7,000 units (700,000 yards/100 maximum number of sales units on the horizontal axis of the graph, and $1,400,000 as the maximum dollar amount on the vertical axis.
3. Prepare a contribution margin income statement showing sales, variable costs, and fixed costs for Product XT at the break-even point.

Answers

Answer:

1a. 3,000 units

1b. $1,050,000

2. See attachment.

3. contribution margin income statement

Sales  ($350 × 7,000 units)                            $2,450,000

Less Variable Cost  ($245 × 7,000 units))     ($1,715,000)

Contribution                                                       $735,000

Less Fixed Costs                                              ( $315,000)

Operating Profit                                                 $420,000

Explanation:

Break-even point (sales units ) = Fixed Cost ÷ Contribution per unit

                                                   = $315,000 ÷ ($350 - $245)

                                                   = 3,000

Break-even point (sales dollars) = Fixed Cost ÷ Contribution Margin Ratio

                                                     = $315,000 ÷ ($105/$350)

                                                     = $1,050,000

A firm's total cost function is given by the equation TC=4000+5Q+10Q and marginal cost is given by the equation MC=5+20Q
(A) Write an expression for each of the following cost concepts:
a. Total Fixed Cost
b. Average Fixed Cost
c. Total Variable Cost
d. Average Variable Cost
e. Average Total Cost
(B) Calculate the values of marginal cost and the costs in (a)-(e) above for Q=0,1,2,3.
(C) Determine the quantity that minimizes average total cost. Demonstrate that the predicted relationship between marginal cost and average cost holds.

Answers

The answer is A because of 5q allowing it to be MC

Following are the calculation to the given question:

[tex]\to TC = 4,000 + 5Q + 10 \ Q2\\\\\to MC = 5 + 20\ Q\\\\[/tex]

For point A)

[tex](a)\ TFC = 4,000\\\\(b)\ AFC = \frac{TFC}{ Q} = \frac{4,000}{ Q}\\\\(c)\ TVC = 5Q + 10\ Q2\\\\(d)\ AVC = \frac{TVC }{Q} = 50 + 10\ Q\\\\(e)\ ATC = \frac{TC }{ Q} = (\frac{4,000}{ Q}) + 50 + 10Q \ \text{Also, ATC = AVC + AFC}\\\\[/tex]

For point B)

TFC remains unchanged at 4,000, regardless of the price of Q.

i)

[tex]\to Q = 0[/tex]

AFC, AVC, and ATC cannot be calculated (division by zero is not possible).

ii)

[tex]Q = 1\\\\AFC =\frac{4,000}{ 1} = 4,000\\\\TVC = (5 \times 1) + (10 \times 1) =5 + 10 = 15\\\\AVC = \frac{TVC}{ Q} = \frac{15}{1} = 15\\\\ATC = 4,000 + 15 = 4,015\\\\MC = 5 + (20 \times 10 = 5 + 20 = 25[/tex]

iii)

[tex]Q = 2\\\\AFC = \frac{4,000}{ 2} = 2,000\\\\TVC = (5 \times 2) + (10 \times 2 \times 2) = 10 + 40 = 50\\\\AVC = \frac{50}{2} = 25\\\\ATC = 2,000 + 25 = 2,025\\\\MC = 5 + (20 \times 2) = 5 + 40 = 45\\\\[/tex]

iv)

[tex]Q = 3\\\\AFC = \frac{4,000}{ 3} = 1,333.33\\\\TVC = (5 \times 3) + (10 \times 3 \times 3) = 15 + 90 = 105\\\\AVC = \frac{105}{3} = 35\\\\ATC = 1,333.33 + 35 = 1,368.33\\\\MC = 5 + (20 \times 3) = 5 + 60 = 65\\\\[/tex]

For point C)

i)

[tex]ATC[/tex] is minimized when [tex]\frac{dATC}{dQ} = 0[/tex]

[tex](- \frac{4,000}{Q2} ) + 10 = 0\\\\\frac{4,000}{Q2} = 10\\\\Q2 = 400\\\\Q = 20\\[/tex]

ii)

Part (B) shows that as MC increases from Q = 0 to Q = 3, ATC decreases, validating the link.

Learn more:

brainly.com/question/15002834

A production department’s beginning inventory cost includes $478,000 of conversion costs. This department incurs an additional $1,047,500 in conversion costs in the month of March. Equivalent units of production for conversion total 770,000 for March.Required:Calculate the cost per equivalent unit of conversion using the weighted-average method.

Answers

Answer: $1.98

Explanation:

Equivalent Units of Production are used when the manufacturers have not completely finished their products for the year. This helps them express it in terms of fully manufactured units.

Using the weighted average method, the cost per equivalent unit is;

= [tex]\frac{Beginning inventory cost + Cost of current production}{Equivalent units of production}[/tex]

= [tex]\frac{478,000 + 1,047,500}{770,000}[/tex]

= $1.98

You have invested 20 percent of your portfolio in Homer, Inc., 40 percent in Marge Co., and 20 percent in Bart Resources. What is the expected return of your portfolio if Homer, Marge, and Bart have expected returns of 2 percent, 18 percent, and 3 percent, respectively?

Answers

Answer:

Expected return = 8.2%

Explanation:

A portfolio is a collection of assets/ investment. The return on a portfolio is the weighted average of all the return of the individual assets weighted according to the percentage of total funds allocated to each assets.

Expected return on portfolio:

E(R) =( Wa*Ra) + (Wb*Rb)  + (Wc*Rc) + Wn*Rn

W= Weight i.e proportion of fund invested in each asset class

Wa = 20%, Wb- 40%, Wc- 20%

Ra-2%, Rb-18%, Rc- 3%

E(R) = (0.2 *2%) + (0.4× 18%) + (0.2*3%) = 8.2%

Expected return = 8.2%

Sam has contracted with Dave to purchase Dave's racing bike, with payment and delivery of the bicycle to be made 10 days after the contract was made. Three days later Sam hears that Dave is going to sell the bike to Gene in three days at a higher price. If Sam really wants the bike, what should he do? Multiple Choice Immediately seek injunctive relief. Immediately sue for specific performance. Immediately sue for compensatory damages. Immediately sue for consequential damages.

Answers

Answer: Immediately seek injunctive relief.

Explanation:

An injunctive relief is an order by the court stopping an action from taking place. From the question, we are told that Sam has contracted with Dave to buy Dave's racing bike, with payment and delivery of the bicycle to be made 10 days after the contract was made.

We are further told that three days later Sam hears that Dave is going to sell the bike to Gene in three days at a higher price. If Sam really wants the bike, he should seek injunctive relief. By doing so, the court will stop Dave from selling the bike to Gene.

Absolute Company has a manufacturing facility in Brooklyn that manufactures robotic equipment for the auto industry. For Year​ 1, Absolute collected the following information from its main production​ line:Actual quantity purchased 200 unitsActual quantity used 110 unitsUnits standard quantity 100 unitsActual price paid $8 per unitStandard price $10 per unitAbsolute isolates price variances at the time of purchase. What is the materials price variance for Year 1?a. $400 favorableb. $400 unfavorablec. $220 favorabled. $220 unfavorable

Answers

Answer:

Direct material price variance= $400 favorable

Explanation:

Giving the following information:

Actual quantity purchased 200 units

Actual price paid $8 per unit

Standard price $10 per unit

To calculate the direct material price variance, we need to use the following formula:

Direct material price variance= (standard price - actual price)*actual quantity

Direct material price variance= (10 - 8)*200

Direct material price variance= $400 favorable

he carrying value of Blossom’s net identifiable assets, including the goodwill, at year-end is $855,000. Prepare Cullumber’s journal entry, if necessary, to record impairment of goodwill.

Answers

Answer:

Goodwill Impairment (Debit)

           Goodwill (Credit)

Explanation:

In case goodwill is impaired, then the entry to record this impairment will be Goodwill Impairment Debit and Goodwill Credit.

By crediting the Goodwill, the account will be reduced. This shows that the business is currently worth less than is accounted for. The Goodwill account is reduced to identify this difference.

The Impairment loss is an expense and must be reflected in the income statement. Therefore, while we reduce Goodwill amount from balance sheet. We record the expense on the income statement, which would mean that the current year profit amount will be reduced.

Huron Company produces a commercial cleaning compound known as Zoom. The direct materials and direct labor standards for one unit of Zoom are given below:
Standard Quantity or Hours Standard Price or Rate Standard Cost
Direct materials 5.50 pounds $ 2.50 per pound $ 13.75
Direct labor 0.50 hours $ 6.50 per hour $ 3.25
During the most recent month, the following activity was recorded:
1. Ten thousand six hundred pounds of material were purchased at a cost of $2.40 per pound.
2. The company produced only 1,060 units, using 9,540 pounds of material. (The rest of the material purchased remained in raw materials inventory.)
3. 630 hours of direct labor time were recorded at a total labor cost of $7,560.
Required:
Compute the materials price and quantity variances for the month. (Indicate the effect of each variance by selecting "F" for favorable, "U" for unfavorable, and "None" for no effect (i.e., zero variance). Input all amounts as positive values. Do not round intermediate calculations.)

Answers

Answer:

Instructions are below.

Explanation:

Giving the following information:

Direct materials 5.50 pounds $ 2.50 per pound.

Actual:

1. 10,600 were purchased for $2.40 per pound.

2. The company produced only 1,060 units, using 9,540 pounds of material.

To calculate the direct material price and quantity variance, we need to use the following formulas:

Direct material price variance= (standard price - actual price)*actual quantity

Direct material price variance= (2.5 - 2.4)*10,600

Direct material price variance= $1,060 favorable

Direct material quantity variance= (standard quantity - actual quantity)*standard price

standard quantity= 1,060*5.5= 5,830

Direct material quantity variance= (5,830 - 9,540)*2.5

Direct material quantity variance= $9,275 unfavorable

Rally Quadcopters plans to sell a standard quadcopter (toy drone) for $45 and a deluxe quadcopter for $65. Rally purchases the standard quadcopter for $35 and the deluxe quadcopter for $45. Management expects to sell two deluxe quadcopters for every three standard quadcopters. The company's monthly fixed expenses are $14,700. How many of each type of quadcopter must Rally sell monthly to breakeven?
To earn $10,500?
First identify the formula to compute the sales in units at various levels of operating income using the contribution margin approach.

Answers

Answer:

Rally must sell 1,080 units of Standard and 720 units of Deluxe

Explanation:

                                                  Standard       Deluxe        Total

Sales price per unit                      $45                $65

Less: Variable cost                      ($35)              ($45)

Contribution Margin per  unit       $10                $20

Sales Mix units  (A)                        $3                  $2                $5

Contribution margin                      $30                $40             $70

Weighted average Contribution                                              $14    

per unit C= B/A

Appointment of fixed cost between standard and deluxe

Total Fixed cost = 14,700

Break even point = Fixed cost / Weighted average Contribution  per unit

= 14,700 / 14

= 1,050

Apportionment of Break even point sales between Standard and deluxe in sales mix ratio (3:2)

Standard = 1,050 * 3/5 = 630

Deluxe = 1,050 * 2/3 = 420

Unit to be sold to get desired profit = Fixed cost + Desired profit / Weighted average Contribution per unit

= (14,700 + 10,500) / 14

= 1,800

Apportionment of Units to be sold to get desired profit between Standard and Deluxe in sales mix ratio (3:2)

Standard = 1,800 * 3/5 = 1,080

Deluxe = 1,800 * 2/5 = 720

To reach target operating income, Rally must sell 1,080 units of Standard and 720 units of Deluxe

what is not a major benefit of co-locating team members from different cultures in one place instead of having a team

Answers

Incomplete question. Here are the options:

A. Short distance to the customer markets

B. Reduced burden from travelling and international meetings

C. Enhanced communications and a sense of community

D. Identical working hours without time zone difference

Answer:

A. Short distance to the customer markets

Explanation:

It is noteworthy to remember we are concerned about what is not a major benefit of co-locating team members from different cultures in one place instead of having a team.

The other benefits like; reduced burden from travelling and international meetings, enhanced communications and a sense of community and having Identical working hours without time zone difference are major in nature as they have a direct impact on cost savings and work efficiency.

A disadvantage of bonds is: Group of answer choices Bonds require payment of periodic interest Bonds require payment of principal Bonds can decrease return on equity Bond payments can be burdensome when income and cash flow are low All of the above

Answers

Answer:

All of the above.

Explanation:

A bond can be defined as a debt or fixed investment security, in which a bondholder (investor or creditor) loans an amount of money to the bond issuer (government or corporations) for a specific period of time. The bond issuer are expected to return the principal (face value) at maturity with an agreed upon interest (coupon), which are paid at fixed intervals.

The disadvantages of bonds are listed below as;

1. Bonds require payment of periodic interest.

2. Bonds require payment of principal.

3. Bonds can decrease return on equity.

4. Bond payments can be burdensome when income and cash flow are low.

If a bank that faces a 10% reserve ratio received a deposit of $50,000 and makes a loan to a customer for $5,000, what is the consequence if the bank then deposits the rest of the funds at the Federal Reserve?

Answers

Answer:

Excess reserve increases by $40,000

Required reserve increases by $5,000

Explanation:

In order to calculate the reserve, we need to multiply the Deposit received by a required reserve ratio.

DATA

Reserve ratio = 10%

Deposit received = $50,000

Loan to customer = $5,000

Solution

Reserve =  Deposit x Required reserve ratio

Reserve = $50,000 x 10%

Reserve = $5,000

After providing a $5,000 loan to the customer and keeping $5,000 as a reserve remaining $40,000 would be deposited in the Federal Reserve.

Which of the following is not a reason why it is important for parties to memorialize their agreements in writing?

a. A party enhances his/her chances of proving that an obligation was undertaken and makes it harder for the other party to deny making the promise.
b. Signing a writing communicates the seriousness of the occasion to the signer.
c. A person's signature on a written contract provides a basis for the contract to be authenticated.
d. Writings are subject to the danger that a person might fabricate terms.

Answers

Answer:

B. singing a writing communicates the seriousness of the occasion to the singer

The reason which is not important for parties to memorialize their agreements in writing is  signing a writing communicates the seriousness of the occasion to the signer. Thus, the correct answer is C.

What is an agreement?

Agreement refers to consent of individual on a particular opinion. When the both parties agree on a concept they will make it in writing. When this agreement enforceable by law it is considered as contract.

The reason it is important top memorialize the agreements in writing are it will act as proof or evidence when formulated in written to be presented in case of obligation.

An agreement will be duly signed by both the parties which shows its authenticity and reliability and avoid any false interpretation of the deal. When the agreement is in writing the violation of terms and conditions is not possible as it clearly mentions the drawbacks of circumstances if any party failed to fulfill the conditions of the agreement.

Therefore, the option C signing a writing communicates seriousness is the appropriate answer.

Learn more about Agreement, here:

https://brainly.com/question/24225827

#SPJ2

What is the present value of a perpetuity that pays you annual, end-of-year payments of $950? Use a nominal rate (monthly compounding) of 7.50%.

Answers

Answer:

The present value of the perpetuity is $12,242.27.

Explanation:

A perpetuity is an annuity that provide cash flow for an infinite period .Examples are Non -redeemable Preference Share.

Present Value (perpetuity) = Payments ÷ Required Rate

But, first change the 7.50 % nominal rate to Annual Effective Rate to match the period of Cash flow.

Effective Rate = (1 + r / m)^m - 1

                       = ( 1 + 0.0750 / 12) ^12 -1

                       = 7.76%

Therefore, Present Value (perpetuity) = $950 ÷  7.76%

                                                              = $12,242.27

At the certain interest rate, present value (PV) is the current value of a future sum of money or stream of cash flows.

The discount rate determines the present value of the cash flows, and the higher the discount rate, the lower the current value of future cash flows.

The present value of the perpetuity is $12,242.27.

A perpetuity is an annuity that payments out during an indefinite period of time. Non-redeemable Preference Share is an example.

Present Value (perpetuity) = [tex]\frac{\text{Payments}}{\text{Required Rate}}[/tex]

However, to match the Working capital period, change a 7.50 percent nominal rate to a Yearly Effective Tax rate.

[tex]\text{Effective Rate} = (1 + \frac{r}{m} )^m - 1= [1 + \frac{0.0750}{12}]^{12} -1= 7.76\%[/tex]

Therefore, Present Value (perpetuity)= [tex]\frac{\$950}{7.76\%} = $12,242.27[/tex]

To know more about the calculations of the present value, refer to the link below:

https://brainly.com/question/15036500

On July 1, 20X1, James and Short formed a partnership. James contributed cash. Short, previously a sole proprietor, contributed property other than cash, including realty subject to a mortgage, which the partnership assumed. Short’s capital account on July 1, 20X1, should be recorded at

Answers

Answer:

James and Short LLC

Short's capital account on July 1, 20X1 should be recorded at the fair value of contributed property minus the mortgage liability, which the partnership assumed.

Explanation:

The fair value of contributed property is the current market value of the contributed property by Short.  It is the market value that will determine how the contributed property can be valued.  The market value assumes that the contributed property is being sold in pieces and not as a whole.  This is why the value is considered a fair basis for recognizing the capital contribution of Short into the partnership.

One characteristic of weekly newspapers is that they usually serve national consumers rather than local consumers. tend to have a larger male readership than daily newspapers. emphasize local news and advertising. have a significantly lower CPM than daily newspapers. do not charge premium rates.

Answers

Answer: emphasize local news and advertising.

Explanation:

One of the main function of the newspaper is to pass message accross to its readership. The newspaper sends information accross to people by informing them about happenings in the local and international scene.

One characteristic of weekly newspapers is that they emphasize local news and advertising. People can advertise on weekly newspapers and gets information across to the local people.

The smartest thing a firm involved in an oligopoly market could do is to cut their prices and capture more of the market share from their competitors.

a) We learned in class that the best move would be to raise prices.

b) We also learned that cutting prices on an elastic demand curve will be a smart way of getting more revenues.

c) Cutting prices is no gaurantee of success. Indeed if the firm does capture more market share and customers, then their costs will go up and it will be harder for them because they will have lower profit margins - if they can earn any profit at all.

d) Both A and C are correct.

Answers

Answer:

Correct Answer:

c) Cutting prices is no gaurantee of success. Indeed if the firm does capture more market share and customers, then their costs will go up and it will be harder for them because they will have lower profit margins - if they can earn any profit at all.

Explanation:

An oligopoly market is a market form wherein a market or industry is dominated by a small group of large sellers. A pure monopoly maximizes profits by producing that quantity where marginal revenue = marginal cost. however, it is much more difficult for an oligopoly to determine at what output it can maximize its profit.

uestion 5
BROOKLYN LTD has developed a new product and is currently considering the marketing and pricing
policy it should employ for this. Specifically, it is considering whether the sales price should be set at Shs.
15,000 per unit or at the higher level of Shs. 24,000 per unit. Sales volume at these two (2) prices is shown
in the following table:
Sales price Shs. 15,000 per Unit
Forecast Sales volume Probability
20,000
0.1
30,000
0,6
40,000
0.3
Sales price Shs. 24,000 per Unit
Forecast Sales volume Probability
8,000
0.1
16,000
0.3
20,000
0.3
24,000
0.3

Answers

Answer:

BROOKLYN LTD

The selling price should be set at Shs. 15,000.  At this price, there are more sales in unit and value than at the selling price of Shs. 24,000.

Explanation:

a) Data and Calculations:

                                            Shs. 15,000   Probability  Expected Sales

Forecasted Sales Volume     20,000              10%         2,000

Forecasted Sales Volume     30,000              60%       18,000

Forecasted Sales Volume     40,000              30%       12,000

Total Expected sales                                                     32,000

Total Sales Value = Shs. 480,000,000 (Shs. 15,000 x 32,000)

                                       Shs. 24,000     Probability  Expected Sales

Forecasted Sales Volume       8,000              10%             800

Forecasted Sales Volume     16,000              30%          4,800

Forecasted Sales Volume    20,000              30%          6,000

Forecasted Sales Volume    24,000              30%          7,200

Total Expected sales                                                      18,800

Total Sales Value = Shs. 451,200,000 (Shs. 24,000 x 18,800)

Conor Airlines Inc. recently issued $50 par value preferred stock that pays a 8.25% dividend rate per year. Yahoo.finance shows that the stock has a beta of 0.97. The current risk-free rate is 2.50% and the market return is 11%. Assuming that CAPM holds, what is the intrinsic value of this preferred stock?

Answers

Answer: $38.39

Explanation:

First calculate the required return according to CAPM;

Required return = Risk free rate + beta ( market return - risk free rate)

= 2.50% + 0.97 ( 11% - 2.50%)

= 10.745%

Then using the Dividend discount model and remembering that there is no growth rate;

Value = Next dividend / ( required return - growth rate)

= (50 * 8.25%) / ( 10.745% - 0)

= 4.125/10.745%

= $38.39

Answer:

$38.29

Explanation:

Ke = Rf+Beta*(Rm-Rf)

Ke=0.0250+0.97*(0.11+0.0250)

Ke=0.10745

Ke=10.75 appr.

Po= Dividend / (Ke-g)

Po= 50*0.0825 / (0.10745 - 0)

Po=4.125/0.10745

Po=38.3899

Po=38.29

Thus, the intrinsiv value of this preferred stock is $38.29

United Apparel has the following balances in its stockholders’ equity accounts on December 31, 2018: Treasury Stock, $650,000; Common Stock, $400,000; Preferred Stock, $1,600,000; Retained Earnings, $1,200,000; and Additional Paid-in Capital, $6,800,000. Required: Prepare the stockholders’ equity section of the balance sheet for United Apparel as of December 31, 2018

Answers

Answer:

United Apparel Balance sheet as of December 31, 2018

Stockholders’ Equity section

Common Stock Capital ............................................$400,000

Preferred Stock Capital.............................................$1,600,000

Additional Paid-in Capital..........................................$6,800,000

Total Paid-in Capital....................................................$8,800,000‬

Retained Earnings.......................................................$1,200,000

Less: Treasury Stock...................................................($650,000)

Total Stockholders Equity..........................................$9,350,000

If the price that determined where marginal revenue equaled marginal cost were below the bottom of the average variable cost curve, then the profit-maximizing, monopolistically competitive firm would

Answers

Answer: c. shut down because it would cost more to produce and sell output than it would to shut down and lose all fixed costs.

Explanation:

The profit maximizing, monopolistically competitive firm maximises profit at the point where marginal revenue equals marginal costs.

If this point is below Average variable costs then that means that the company is not making enough to cover its variable costs. Should this be the case then the company should shutdown operations because variable costs are only there when the company is producing. If they shutdown then they will no longer incur them which would be the cheaper option.

They would take losses on the fixed costs but these have already been incurred so it would be better to lose the fixed costs than continue to make losses on variable costs.

Care Foundation is a voluntary health and welfare organization funded by contributions from the general public. In its Statement of Activities, the annual provision for depreciation should:

Answers

Question options:

A) Not be included.

B) Be included as an element of support.

C) Be included as an element of changes in fund balances.

D) Be included as an element of expense.

Answer:

D) Be included as an element of expense

Explanation:

Care foundation is a voluntary health and welfare organization funded by contributions from the public and therefore is a non-profit organization. Non profit organizations use statement of activities and not income statements used by for profit organizations in reporting revenue and expenses for the year. In the case of non profit organizations, statement of activities are reported as statement of expenses for the year.

Under GASB, direct expenses are expenses that can be linked to a program, department or activity and therefore can be directly linked to that function. Depreciation is a direct expense for non profit accounts and should be charged as expense for the relevant year based on the function of the capital asset it can be traced to. For example a capital asset that can be linked to a particular function should charge it's depreciation expenses as direct expenses based on its functions

Fallow Corporation has two separate profit centers. The following information is available for the most recent year: West Division East Division Sales (net) $ 410,000 $ 560,000 Salary expense 47,000 61,000 Cost of goods sold 143,000 259,000 The West Division occupies 10,250 square feet in the plant. The East Division occupies 6,150 square feet. Rent, which was $ 82,000 for the year, is an indirect expense and is allocated based on square footage. Compute operating income for the West Division.

Answers

Answer:

$168,750

Explanation:

The data below are extracted from the above question.

West division

Sales (S) = $410,000

Salary expense (E) = $47,000

Cost of goods sold (C) = $143,000

Proportional rent (R) = $82,000 % of square footage

Area of the division = 10,250 square feet.

Total area of both division = 10,250 + 6,150

= 16,400 square feet

Therefore, the operating income (I) for the West Division is given by the amount of sales minus salary expenses , cost of goods sold and rent.

I = S - E - C - R

= $410,000 - $47,000 - $143,000 - (82,000 × 10,250 / 16,400)

= $220,000 - $51,250

= $168,750

The yearly operating income for Fallow's Corporation West Division is $168,750.

Suppose that the value of an investment in the stock market has increased at an average compound rate of about 5% since 1912. It is now 2016. a. If someone invested $1,000 in 1912, how much would that investment be worth today?

Answers

Answer:

FV= $159,840.60

Explanation:

Giving the following information:

Initial investment= $1,000

Number of years= 2016 - 1912= 104

Interest rate= 5%

To calculate the value of the investment today, we need to use the following formula:

FV= PV*(1+i)^n

FV= 1,000*(1.05^104)

FV= $159,840.60

Imagine you want to use conflict in a positive way. You decide to create a sense of competition among your team members. Which of these tactics could create competition?]

Answers

Answer:

a. Acknowledge top performers in the company newsletter.

Explanation:

Conflict among group members could be used for improved results by enhancing the dispute in a constructive manner. This can be achieved by recognizing and rewarding the best performers accordingly.

Therefore according to the given situation, for deciding a sense of competition you need to acknowledge the top performance in the newsletter of the company so that the employees gots motivated that results in their coming better job opportunities

Hence,  the correct option is a

Which is the first step toward initiating efficient and effective international business negotiations:

Answers

Answer: Selecting an appropriate negotiation team

Explanation:

The first step toward initiating efficient and effective international business negotiations is selecting an appropriate negotiation team.

When an appropriate negotiation team has been selected to negotiate on behalf of a particular company, negotiation becomes easier and are more feasible and both parties can agree on a particular stance.

You purchased 1,000 shares of stock in Natural Chicken Wings, Inc., at a price of $43.37 per share. Since you purchased the stock, you have received dividends of $.95 per share. Today, you sold your stock at a price of $46.62 per share. What was your total percentage return on this investment?

Answers

Answer:

9.68%

Explanation:

Percent Return on Investment is calculated as Net Profit / Cost of Investment x 100

Net Profit= $46,620 (1,000 x $46.62 per share) + $950 (1,000 x $.95 per share) - $43,370 (1,000 x $43.37 per share) = $4,200

Cost of Investment= $43,370 (1,000 x $43.37 per share)

Percent Return on Investment=  $4,200 / $43,370 x 100 = 9.68%

Instruments had retained earnings of at December​ 31, . Net income for totaled ​, and dividends declared for were . How much retained earnings should report at December​ 31, ​?

Answers

Answer:

B. $ 490,000

Explanation:

According to the given situation, the computation of retained earning in the year end is shown below:-

Ending retained earning = Beginning Retained Earnings + Net Income for the year - Dividend

= $360,000 + $180,000 - $50,000

= $490,000

Therefore for computing the ending retained earning we simply applied the above formula.

Two investment advisors are comparing performance. Advisor A averaged a 20% return with a portfolio beta of 1.5 and Advisor B averaged a 15% return with a portfolio beta of 1.2. If the T-bill rate was 5% and the market return during the period was 13%, which advisor was the better stock picker?

Answers

Answer:

Advisor A

Explanation:

t bill rate = 0.05

market rate = 0.13

the beta of the market is always 1

the rate of return= 0.05 + (0.13 - 0.05) x 1

= 0.13

which is 13%

this is for advisor A.

with a return of 20% and 1.5 beta

0.05 + ( 0.20 - 0.05) x 1.5

= 27.5% for advisor b

when the return is 15% and beta is 1.2

0.05 + (0.15 - 0.05) x 1.2

= 17%

Therefore advisor a is better

Other Questions
You own a stock portfolio invested 30 percent in Stock Q, 25 percent in Stock R, 25 percent in Stock S, and 20 percent in Stock T. The betas for these four stocks are .95, 1.12, 1.13, and 1.30, respectively. What is the portfolio beta? (Do not round intermediate calculations and round your answer to 2 decimal places, e.g., 32.16.) Based on predicted production of 17,000 units, a company anticipates $255,000 of fixed costs and $216,750 of variable costs. The flexible budget amounts of fixed and variable costs for 15,000 units are (Do not round intermediate calculations): All of the following are true statements except _____. |93| = 93 |93| = 93 |93| = 93 |93| = 93 An employee produces 10 parts during a shift in which he made $90. What is the labor content of the product? $5 $900 $9 $0.111 $6 Find the measure of the remote exterior angle. mx=(4n18)my=(n+9)mz=(1515n) A. 71 B. 16 C. 100 D. 46 Matilda has 16 3/4 hours to finish 3 consulting projects. How much time may she spend on each project, if she plans to spend the same amount of time each?A. 5 6/7B. 5 3/7C. 5 9/11D. 5 7/12 Find the odd one out a)Planets moving on it's axis,Strings of guitar (being played),Motion of a ferry wheel,The vehicles moving on a straight road. b)Motion of the moon around the earth,Motion of the earth around the sun,Motion of a merry-go-round,Heart-beat in a healthy person. c)Motion of a bullet fired from gun,Motion of a football player in the ground,Motion of a vehicle on a straight road,Motion of an apple falling from a tree An investment offers a total return of 12.0 percent over the coming year. Janice Yellen thinks the total real return on this investment will be only 6.0 percent. What does Janice believe the inflation rate will be over the next year? 5 . pls ANSWER This question. very urgent. A baby weighs 100 ounces. Find the baby's weight in pounds and ounces. The triangle shown on the graph above is rotated 90 degrees clockwise about the original to form triangle PQR which of the following are the (x,y) coordinates of the point P Find the sum. 1. -7+(-5) O-12 O-2 0 2 0 12 please help me. I been stuck on this question for a long time :( El siguiente diagrama A, B, C, D, E, F denotan islas, y las lneas de unin son puentes. El hombre empieza en A y camina de isla en isla. El hombre no puede cruzar el mismo puente dos veces. Hallar el nmero de maneras que puede hacer su recorrido antes de almorzar.A-B-C-D E-FA esta conectado a B, B a C y C a D. B est conectado a E, C esta conectado a F y hay una linea que conecta E y C HELP, ASAP, 100 ptsRead the passage below. Then answer the question that follows using information and evidence from the text.That on the first day of January, in the year of our Lord one thousand eight hundred and sixty-three, all persons held as slaves within any State or designated part of a State, the people whereof shall then be in rebellion against the United States, shall be then, thenceforward, and forever free; and the Executive Government of the United States, including the military and naval authority thereof, will recognize and maintain the freedom of such persons, and will do no act or acts to repress such persons, or any of them, in any efforts they may make for their actual freedom ...What is one question this passage leaves unanswered? Describe another source that might help you answer that question. For which of the following compound inequalities is there no solution? A. 3m - 12 > 30 and -6m >= 24 B. -6m >= 12 and m + 5 -18 C. -5m < 20 and 6m > -18D. -4m - 10 = 22>= is greater than or equal to We discussed the different types of intermolecular forces in this lesson, which can affect the boiling point of a substance. 1. Which of these has the highest boiling point?A) ArB) KrC) XeD) Ne2. Which substance has the highest boiling point?A) CH4B) HeC) HFD) Cl2 Select all of the numbers that are correctly written in scientific notation. 10.8\times10^{-3}10.810 3 0.54\times10^60.5410 6 1\times10^{-4}110 4 7.6\times10^{2.5}7.610 2.5 9.8\times10^59.810 5 The following data regarding purchases and sales of a commodity were taken from the related perpetual inventory account:June 1Balance 25 units at $606 Sale 20 units8 Purchase 20 units at $6116 Sale 10 units20 Purchase 20 units at $6223 Sale 25 units30 Purchase 15 units at $63Required:Calculate the cost of the ending inventory at June 30, using (a) the first-in, first-out (FIFO) method and (b) the last-in, first-out (LIFO) method. Identify the quantity, unit price, and total cost of each lot in the inventory. What is the magnitude of the applied electric field inside an aluminum wire of radius 1.4 mm that carries a 4.5-A current